Lưu trữ

Archive for the ‘Phương trình nghiệm nguyên’ Category

Number theory 30

(Switzerland National Olympiad 2007) Tìm mọi số tự nhiên a,b thỏa mãn \dfrac{a^3+1}{2ab^2+1} là số nguyên.

Lời giải. Xem tại đây.

Number theory 29

Giải phương trình nghiệm nguyên không âm 33^x+31=2^y.

Lời giải. Xem tại đây.

Number theory 28

Giải phương trình nghiệm nguyên không âm 11^a5^b-3^c2^d=1.

Lời giải. Xem tại đây.

Number theory 22

(Vietnam VMO 2004) Tìm tất cả các số nguyên dương (x,y,z) sao cho (x+y)(xy+1)=2^z.

Lời giải 1. Đặt x+y=2^m,xy+1=2^n với m,n \in \mathbb{N}^*. Vì xy+1-(x+y)=(x-1)(y-1) \ge 0 nên m \le n. Nếu m=1 thì x+y=1 nên x=y=1. Khi đó z=2.

Nếu m=2 thì x+y=4 suy ra x=1,y=3 hoặc x=3,y=1. Khi đó z=4.

Nếu m \ge 3. Khi đó thì \begin{cases} x+y=2^m \\ xy=2^n-1 \end{cases}. Theo định lý Vieta ta suy ra x,y là hai nghiệm nguyên dương của phương trình

\begin{array}{l} a^2-2^ma+2^n-1=0, \qquad (1) \\ \Delta = 2^{2m}-4(2^n-1)=2^{2m}-2^{n+2}+4 \end{array}

Để (1) có nghiệm nguyên thì \Delta là số chính phương, do đó ta đặt 2^{2m-2}-2^n+1=p^2 với p \in \mathbb{N}.

Đặt p=2^{m-1} - \alpha \; (\alpha \in \mathbb{N}, \; \alpha \le 2^{m-1}) thì phương trình trở thành

1-2^n= \alpha^2- 2^m \alpha \Leftrightarrow 2^m \left( \alpha - 2^{n-m} \right) = \alpha^2-1 \qquad (2)

Dễ thấy rằng \alpha lẻ, đặt \alpha= 2^k \cdot q+1 với k,q \in \mathbb{N}, k,q \ge 1, 2 \nmid q. Khi đó

(2) \Leftrightarrow 2^m \left( \alpha - 2^{n-m} \right) = 2^{k+1} \cdot q \cdot \left( 2^{k-1} \cdot q+1 \right) \qquad (3)

\blacktriangleright Nếu k=1 thì \alpha=2q+1. Do đó (2) \Leftrightarrow 2^m \left( \alpha - 2^{n-m} \right) = 4q(q+1). Tiếp tục đặt q=2^o-1 với o \in \mathbb{N}, o \ge 1 thì ta có 2^m \left( \alpha - 2^{n-m} \right) = 2^{o+2} \left( 2^o-1 \right). Từ đây ta suy ra m=2+o2 \left( 2^{o}-1 \right) - 2^{n-m}+1= 2^o-1. Do đó 2^o-2^{n-m}=0. Vậy o=n-m.

Quay lại với (3), ta có \alpha= 2q+1=2 \left( 2^o-1 \right) +1=2^{o+1}-1. Thay vào (3) ta được

2^m \left( 2^{n-m}-1 \right) = 4 \cdot \left( 2^{n-m}-1 \right) \cdot 2^{n-m} \qquad (4)

Nếu n=m thì x+y=xy+1 \Leftrightarrow (x-1)(y-1)=0. Không mất tính tổng quát, giả sử x=1 thì phương trình ban đầu tương đương với (y+1)^2=2^z. Vậy z=2l, y=2^l-1 với l \in \mathbb{N}^*.

Nếu n>m thì (4) \Leftrightarrow 2^m=2^{n-m+2} \Leftrightarrow 2m=n+2. Ta suy ra (x+y)^2=4xy+4 \Leftrightarrow (x-y)^2=4 nên x-y=2 hoặc y-x=2. Không mất tính tổng quát, giả sử x \ge y. Do đó x=y+2. Thay vào phương trình ban đầu ta được 2(y+1)^3=2^z \Leftrightarrow (y+1)^3=2^{z-1}. Vậy z-1=3g,y=2^g-1 với g \in \mathbb{N}^*.

\blacktriangleright Nếu k \ge 2 thì (3) \Rightarrow m=k+1

\begin{aligned} \alpha - 2^{n-m} = 2^{m-2}q^2+q & \Leftrightarrow 2^{m-1}q+1-2^{n-m}=2^{m-2}q^2+q \\ & \Leftrightarrow 2^{m-2} (2q-q^2) = 2^{n-m}+q-1 \end{aligned}

Với q \ge 3 thì ta có VT<0<VP. Vậy q=1. Khi đó thì 2^{m-1}-2^{m-2}-2^{n-m}=0 \Leftrightarrow 2^{m-2}=2^{n-m}. Vậy 2m=n+2. Làm tương tự như trường hợp n>m.

Vậy, phương trình có nghiệm nguyên dương (x,y,z)= \left( 1,2^l-1,2l \right); \left( 2^l-1,1,2l \right); \left( 2^l+1,2^l-1,3l+1 \right), \left( 2^l-1,2^l+1,3l+1 \right) với l \in \mathbb{N}^*.

Lời giải 2. Ta có x+y=2^m,xy+1=2^{z-m} với z \ge 2m, m \in \mathbb{N}^*. Không mất tính tổng quát, giả sử x \le y.

Nếu x=1 thì y=2^m-1, z=2m.

Nếu x,y \ge 3 thì ta có 2^m|x^2-1x^2-1=x(x+y)-(xy+1). Hiển nhiên rằng \gcd (x-1,x+1)=2 nên 2^{m-1}|x-1 hoặc 2^{m-1}|x+1. Nhưng 0<x-1<\frac 12 (x+y)-1<2^{m-1} Vậy 2^{m-1}|x+1. Ta suy ra x+1 \ge 2^{m-1}.

Nếu y=x thì 2x(x^2+1) là lũy thừa của 2, mâu thuẫn.

Nếu y=x+1 mâu thuẫn vì x,y cùng lẻ. Vậy y \ge x+2. Do đó x+1 \le \frac 12 (x+y-2)+1=2^{m-1}. Vậy x+1=2^{m-1}.

Vậy phương trình có nghiệm nguyên dương (x,y,z)=(1,2^m-1,2m),(2^m-1,1,2m),(2^{m-1}-1,2^{m-1}+1,3m-2),(2^{m-1}+1,2^{m-1}-1,3m-2) với m \ge 3 là số tự nhiên.

Number theory 19

Tìm nghiệm nguyên của phương trình: x^3+10x-1=y^3+6y^2.

Lời giải. Đặt x=y+b với b \in \mathbb{Z}. Ta có

\begin{aligned} (1) & \Leftrightarrow y^3+3y^2b+3yb^2+b^3+10y+10b-1=y^3+6y^2 \\ & \Leftrightarrow y^2(3b-6)+y(3b^2+10)+b^3+10b-1=0 \qquad (1) \\ \Delta & = (3b^2+10)^2-(12-24b)(b^3+10b-1) \ge 0 \\ & = -3b^4+24b^3-60b^2+252b+76 \\ & = 1399- 3(b^2-4b)^2-3(2b-21)^2 \ge 0 \end{aligned}

Do đó (b^2-4b)^2+(2b-21)^2 \le 466. Nhận thấy (2b-21)^2 \le 466 nên 0 \le b \le 21.

Theo phương trình ban đầu thì x,y khác tính chẵn lẻ nên b lẻ.

Nếu b=1 thì (1) \Leftrightarrow -3y^2+13y+10=0 \Leftrightarrow y=5 \Rightarrow x=6.

Nếu b=3 thì (1) \Leftrightarrow 3y^2+37y+56=0, không có nghiệm nguyên.

Nếu b=5 thì (1) \Leftrightarrow 9y^2+85y+174=0 \Leftrightarrow y=-3. Khi đó x=2.

Nếu b=7 thì (1) \Leftrightarrow 15y^2+157y+412=0, vô nghiệm.

Nếu b=11 thì (1) \Leftrightarrow 27y^2+373y+1440=0, vô nghiệm.

Nếu b=13 thì (1) \Leftrightarrow 33y^2+517y+2326=0, vô nghiệm.

Nếu b=15 thì (1) \Leftrightarrow 39y^2+685y+3524=0, vô nghiệm.

Nếu b=17 thì (1) \Leftrightarrow 45y^2+877y+5082=0, vô nghiệm.

Nếu b=19 thì (1) \Leftrightarrow 51y^2+1093y+7048=0, vô nghiệm.

Nếu b=21 thì (1) \Leftrightarrow 57y^2+442y+9470=0, vô nghiệm.

Vậy phương trình có nghiệm nguyên \boxed{(x,y)=(6,5),(2,-3)}.

Nhận xét. Một ví dụ điển hình cho kĩ thuật giảm bậc trong giải phương trình nghiệm nguyên.

Number theory 18

(Đề thi chọn đội tuyển HSG tỉnh Nghệ An năm 2011) Tìm tất cả các số nguyên dương x,y thỏa mãn

x^{2010}+x^{2009}+ \cdots + x+2= y^5

Lời giải. Xét bổ đề: Nếu p,q là số nguyên tố, a là số nguyên dương lớn hơn 1 thoả mãn q| \dfrac{a^p-1}{a-1} thì hoặc p=q hoặc q \equiv 1 \pmod{p}.

Chứng minh. Gọi k là số nguyên dương nhỏ nhất thoả mãn x^k \equiv 1 \pmod{q}. Theo đề bài ta suy ra x^p \equiv 1 \pmod{q}. Vậy k|p. Do đó k=1 hoặc k=p.

Nếu k=1 thì x \equiv 1 \pmod{q}. Do đó x^{p-1}+x^{p-2}+ \cdots + 1 \equiv p \pmod{q}. Do đó p \equiv 0 \pmod{q} hay p=q.

Nếu k=p. Theo định lý Fermat nhỏ ta lại có x^{q-1} \equiv 1 \pmod{q} nên p|q-1. Vậy q \equiv 1 \pmod{p}.

Quay lại bài toán. Phương trình tương đương với \dfrac{x^{2011}-1}{x-1}=y^5-1. Nhận thấy 2011 nguyên tố.

Gọi p là một ước nguyên tố bất kì của y^5-1=(y-1)(y^4+y^3+y^2+y+1). Ta suy ra p=2011 hoặc p \equiv 1 \pmod{2011}. Vậy các ước của y^5-1 hoặc chia hết cho 2011 hoặc chia 20111

Nếu 2011|y-1. Khi đó y \equiv 1 \pmod{2011} với k \in \mathbb{N}^*, điều này mâu thuẫn vì y^4+y^3+y^2+y+1 \equiv 4 \pmod{2011}.

Nếu y-1 \equiv 1 \pmod{2011}. Khi đó y \equiv 2 \pmod{2011} nên y^4+y^3+y^2+y+1 \equiv 31 \pmod{2011}, mâu thuẫn.

Vậy phương trình không có nghiệm nguyên dương.

Tổng quát. Nếu p nguyên tố lớn hơn 3, n là số nguyên dương sao cho các số p-1,p,n,n+1 từng đôi một không có ước số chung lớn hơn 2. Chứng minh rằng phương trình sau không có nghiệm nguyên

x^{p-1}+x^{p-2}+ \cdots + x+2= y^{n+1}

Nhận xét. Bài toán này giống với bài toán trong IMO Shortlist 2006.

Number theory 17

Tháng Tám 29, 2013 1 Bình luận

(Balkan MO 2005) Tìm tất cả các số nguyên tố p thỏa mãn p^2-p+1 là lập phương của một số tự nhiên.

Lời giải. Đặt p^2-p+1=a^3 với a \in \mathbb{N}^*. Phương trình tương được với p(p-1)=(a-1)(a^2+a+1) \qquad (1)

Nếu p|a-1 thì a=pk+1 với k \in \mathbb{N}. Khi đó (1) \Leftrightarrow p^2-p+1=p^3k^3+3p^2k^2+3pk+1

Với k \ge 2 thì VT<VP, mâu thuẫn. Vậy 0 \le k \le 1.

Với k=0 thì p^2-p+1=1 \Leftrightarrow p(p-1)=0, mâu thuẫn.

Với k=1 thì p^3-2p^2+4p=0 \Leftrightarrow (p-1)^2+2=0, mâu thuẫn.

Vậy p \nmid a-1. Do đó p| a^2+a+1 \qquad (2).

Lại có a-1|p(p-1)\gcd (p,a-1)=1 nên a-1|p-1. Đặt p=(a-1)o+1 với o \in \mathbb{N}. Khi đó từ (2) ta suy ra \dfrac{a^2+a+1}{(a-1)o+1} là số nguyên dương. Hay

\dfrac{a^2o+ao+o}{ao-o+1}= \dfrac{ a(ao-o+1)+2(ao-o+1)+3o-2-a}{ao-o+1}= a+2+ \dfrac{3o-2-a}{ao-o+1} \qquad (3)

nguyên dương. Do đó ta phải có |3o-2-a| \ge ao-o+1.

Trường hợp 1. Nếu 3o-2-a \ge ao-o+1 \Leftrightarrow o(4-a) \ge a+3.

Nếu a \ge 4 thì o(4-a)<a+3, mâu thuẫn. Vậy 2 \le a \le 3. Nếu a=3 thì phương trình ban đầu tương đương với p(p-1)=26, mâu thuẫn. Nếu a=2 thì phương trình ban đầu tương đương với p(p-1)=7, mâu thuẫn.

Trường hợp 2. Nếu 2+a-3o \ge ao-o+1 \Leftrightarrow o(2+a) \le a+1, mâu thuẫn vì a+1>a+2.

Trường hợp 3. Nếu 3o-2-a=0 \Leftrightarrow a=3o-2. Khi đó p=3o(o-1)+1a^2+a+1=(3o-2)^2+(3o-2)+1=9o^2-9o+3. Do đó \frac{a^2+a+1}{p}=3. Ta có

(1) \Leftrightarrow p-1=3(a-1) \Leftrightarrow (o-1)(o-3)=0 \Leftrightarrow \left[ \begin{array}{l} o=1 \\ o=3 \end{array} \right.

Nếu o=1 thì p=a, hoàn toàn mâu thuẫn vì khi đó p^2+p+1=p^3 suy ra p(p^2-p-1)=1.

Nếu o=3 thì p=3a-2. Khi đó (1) \Leftrightarrow 9a^2-15a+7=a^3 \Leftrightarrow \left[ \begin{array}{l} a=7 \\ a=1 \end{array} \right.. Thử lại thấy a=7 thoả mãn. Khi đó p=19.

Vậy \boxed{p=19} là đáp án bài toán.

Number theory 15

(Junior Balkan MO 2000) Tìm các cặp số nguyên dương n thỏa mãn 3^n+n^2 là số chính phương.

Lời giải. Đặt n^2+3^n=a^2 với a \in \mathbb{N}^*n=3^p \cdot q với p,q \in \mathbb{N}, \; \gcd (3,q)=1. Vì n \ge 1 nên q \ge 1. Do đó 3^p \cdot q > 2p.

Ta suy ra 3^{2p} |n^2+3^n. Do đó 3^{p} |a. Đặt a=3^p \cdot k với k \in \mathbb{N}^*, \; \gcd (k,3)=1.

Khi đó phương trình trở thành q^2+3^{h}=k^2 với h=3^p \cdot q-2p \in \mathbb{N}^*, \; \gcd (3,q)=1, \; \gcd (k,3)=1.

Phương trình tương đương với \left( k-q \right) \left( k+q \right) = 3^h

Đặt k-q=3^a, \; k+q=3^b với a,b \in \mathbb{N} , \; a<b, \; a+b=h. Khi đó 2q=3^a \left( 3^{b-a}-1 \right).

\gcd (q,3)=1 nên a=0. Do đó 2q=3^h-1 hay 2q= 3^{3^p \cdot q-2p}-1 \qquad (1).

Ta sẽ chứng minh bằng quy nạp hai bổ đề sau:

Bổ đề 1. Với mọi p \in \mathbb{N} thì 3^p \ge 2p+1

Chứng minh. Với p=0 thì BĐT đúng. Giả sử BĐT đúng với p=l \in \mathbb{N} thì 3^l \ge 2l+1.

Ta sẽ đi chứng minh BĐT cũng đúng với p=l+1, tức 3^{l+1} \ge 2(l+1)+1.

Thật vậy thì 3^{l+1} \ge 3 \cdot (2l+1) \ge 2(l+1)+1.

Vậy BĐT được chứng minh. Dấu đẳng thức xảy ra khi và chỉ khi p=0 hoặc p=1.

Bổ đề 2. Với mọi q \in \mathbb{N}^* thì ta luôn có 3^p \cdot q \ge 2p+q (trong đó p \in \mathbb{N}).

Chứng minh. Với q=1 thì 3^p \ge 2p+1, đúng với p \in \mathbb{N} theo bổ đề 1. Giả sử BĐT đúng với q=l \in \mathbb{N}^*. Khi đó 3^p \cdot l \ge 2p+l.

Ta cần chứng minh BĐT cũng đúng với q=l+1, tức 3^p \cdot (l+1) \ge 2p+ (l+1). Thật vậy, 3^p \cdot (l+1) \ge 2p+l +3^p \ge 2p+l+1, đúng.

Dấu đẳng thức xảy ra khi q=1,p=0 hoặc q=1,p=1.

Quay lại (1). Theo bổ để 2 ta có 3^p \cdot q-2p \ge q \Rightarrow 2q+1 \ge 3^q-1. Tuy nhiên theo bổ đề 1 nên ta suy ra q=1. Khi đó 3^p=2p+1. Áp dụng một lần nữa bổ để 1 thì dẫn đến p=0 hoặc p=1. Vậy n=1 hoặc n=3.

Vậy số tự nhiên n cần tìm là \boxed{ n \in \{ 1;3 \}}.

Number theory 14

Giải phương trình nghiệm nguyên dương x^{2}y+x+y=xy^{2}z+yz+7z.

Lời giải. Vì y,x \in \mathbb{N}^* nên xy^2+y+7>0. Phương trình tương đương với z= \dfrac{x^2y+x+y}{y^2x+y+7}.

Bài toán được đưa về dạng tìm x,y nguyên dương sao cho z nguyên dương. Ta thấy z nguyên dương thì

y \cdot z= \dfrac{x^2y^2+xy+y^2}{y^2x+y+7}= x+ \dfrac{y^2-7x}{y^2x+y+7}

nguyên dương. Khi đó \dfrac{y^2-7x}{y^2x+y+7} nguyên. Do đó |y^2-7x| \ge y^2x+y+7.

\blacktriangleright Trường hợp 1. Nếu y^2>7x thì y^2-7x \ge y^2x+y+7 \Leftrightarrow (y^2+7)(1-x) \ge 14+y. Vì x,y \in \mathbb{N}^* nên (y^2+7)(1-x) \le 0 còn 14+y>0, mâu thuẫn.

\blacktriangleright Trường hợp 2. Nếu y^2<7x thì 7x-y^2 \ge y^2x+y+7 \Leftrightarrow (7-y^2)(x+1) \ge 14+y.

Với y^2 >7 thì (x+1)(7-y^2) <014+y>0, mâu thuẫn.

Vậy y^2 \le 7 \Rightarrow y^2 \in \{ 1 ; 4 \} \Rightarrow y \in \{ 1;2 \}.

Nếu y=1 thì z=\dfrac{x^2+x+1}{x+8}= x-7+ \dfrac{57}{x+8} là số nguyên dương khi x+8 \mid 57. Do x+8 \ge 9 nên x+8 \in \{ 19;57 \} \Rightarrow x \in \{ 11;49 \}.

Nếu y=2 thì z= \dfrac{2x^2+x+2}{4x+9} \Rightarrow 16z= 4x-7+ \dfrac{79}{4x+9} là số nguyên dương thì 4x+9 \mid 79.

Lại có 4x+9 \ge 13 nên 4x+9=79 \Rightarrow x= \frac{35}{2}, mâu thuẫn với điều kiện $x$ nguyên dương.

\blacktriangleright Trường hợp 3. Nếu y^2=7x thì để x \in \mathbb{N}^* thì x=7t^2 với t \in \mathbb{N}^*. Khi đó y=7t.

Cũng vì y^2=7x nên y \cdot z=x \Rightarrow z=t.

Vậy phương trình có nghiệm nguyên dương \boxed{ (x,y,z)=(11;1;7),(49;1;43), (7t^2,7t,t) } với mọi t \in \mathbb{N}^*.

Number theory 13

(Mathematical Reflection Magazine Issue 4- 2013) Giải phương trình nghiệm nguyên dương 10^n-6^m=4n^2.

Lời giải. Nhận thấy m=n=1 là một nghiệm của phương trình.

Thử với n=2,3 thì không thỏa mãn. Với n \ge 4, ta có bổ đề sau:

Bổ đề 1. Nếu a \in \mathbb{N}, \; a \ge 2 thì 10^a>6^a+4a^2.

Bổ đề này dễ chứng minh bằng quy nạp. Do vậy, từ bổ đề ta suy ra 6^m>6^n \Rightarrow m>n \ge 4.

Trường hợp 1. Nếu n lẻ thì 4n^2 \equiv 4 \pmod{16}10^n \equiv 0 \pmod{16} (vì n \ge 4). Do đó 6^m \equiv 12 \pmod{16} \Rightarrow 6^m=16k+12=4(4k+3). Ta suy ra m=2. Tuy nhiên m=2 thì 6^m=6^2 \equiv 4 \pmod{16}, mâu thuẫn.

Trường hợp 2. Nếu n chẵn thì n=2n_1 với n_1 \in \mathbb{N}, \; n_1 \ge 2. Mặt khác, ta có nhận xét sau:

Nhận xét. Nếu a \in \mathbb{Z} thì a^2 \equiv 0,1,2,4 \pmod{7}.

Từ nhận xét ta suy ra 4n^2 \equiv 0,1,2,4 \pmod{7}.

Khả năng 1. Nếu 3|n_1 thì 3|4n^23|6^m nên 3|10^n, mâu thuẫn.

Khả năng 2. Nếu n_1=3k+1 thì theo định lý Fermat nhỏ ta có 10^n=10^{6k+2} \equiv 2 \pmod{7}.

+) Với m chẵn thì 6^m \equiv 1 \pmod{7} \Rightarrow 4n^2 \equiv 1 \pmod{7} (có thể xảy ra).

+) Với m lẻ thì 6^m \equiv 6 \pmod{7} \Rightarrow 4n^2 \equiv 3 \pmod{7}, mâu thuẫn.

Khả năng 3. Nếu n_1=3k+2 thì 10^n=10^{6k+4} \equiv 4 \pmod{7}.

+) Với m chẵn thì 4n^2 \equiv 3 \pmod{7}, mâu thuẫn.

+) Với m lẻ thì 4n^2 \equiv 5 \pmod{7}, mâu thuẫn.

Vậy ba khả năng trên ta suy ra m=2m_1 với m_1 \in \mathbb{N}, \; m_1 \ge 2.

Như vậy (1) \Leftrightarrow \left( 10^{n_1}-6^{m_1} \right) \left( 10^{n_1}+6^{n_1} \right) = 16n_1^2 \qquad (2)

m>n nên (2) \Leftrightarrow 2^{2n_1} \left( 5^{n_1}-2^{m_1-n_1} \cdot 3^{m_1} \right) \left( 5^{n_1}+2^{m_1-n_1} \cdot 3^{m_1} \right)= 16n_1^2

Đặt n_1=2^q \cdot k với q,k \in \mathbb{N},k lẻ. Khi đó phương trình trở thành

\left( 5^{n_1}-2^{m_1-n_1} \cdot 3^{m_1} \right) \left( 5^{n_1}+2^{m_1-n_1} \cdot 3^{m_1} \right)= \dfrac{2^{2q+4}}{2^{2^{q+1} \cdot k}} \cdot k^2

m>n nên các số 5^{n_1}-2^{m_1-n_1} \cdot 3^{m_1}, 5^{n_1}+2^{m_1-n_1} \cdot 3^{m_1} đều là số lẻ. Do đó 2^{2q+4}=2^{2^{q+1} \cdot k} \Rightarrow 2q+4=2^{q+1} \cdot k \Rightarrow q+2=2^q \cdot k.

Ta sẽ đi chứng minh bằng quy nạp: Với mọi q \ge 1,k \ge 3 thì 2^q \cdot k>q+2.

Do đó chỉ có thể k=1. Vậy \left( 5^{n_1}-2^{m_1-n_1} \cdot 3^{m_1} \right) \left( 5^{n_1}+2^{m_1-n_1} \cdot 3^{m_1} \right) =1, điều này hiển nhiên mâu thuẫn do 5^{n_1}+2^{m_1-n_1} \cdot 3^{m_1}>1.

Vậy phương trình có nghiệm nguyên duy nhất \boxed{(m,n)=(1,1)}.

File lời giải: J276Issue 4